SBI PO (Main) Exam Paper - 2018 "Reasoning & Computer Aptitude"

SBI PO (Main) Exam Paper - 2018 "Reasoning & Computer Aptitude"

Direction (1-5): Study the following information carefully and answer the questions given below:
Ten people live in three buildings namely A, B and C. Building A is to the immediate left of building B whereas building C is to the second right of building A. A maximum of four people live in a building.
Note: Immediately above or below implies that the person is not in the same building.
As many floors are there below the floor, in which T lives, as it is above the floor in which Y lives. There are four floors above the floor in which S lives. S doesn’t live in building B but lives at floor number 5. Z lives in that floor number, which is the sum of the floor number in which R and S live. The sum of the floor number in which P and T lives is 3 and both lives in different building. R lives in building B and lives at the topmost floor. T and Y live in same building but they do not live in building A. The sum of the number of the floor in which T and V lives is equal to the floor number in which Y live. T lives in the even numbered floor. Equal number of floors are there below the floor in which Q lives and above the floor in which W lives. There are two floors between W and Q. U lives on that floor number which is immediately below the floor number in which V lives. Only Two people live in Building C. S and U live in same building. R lives immediately two floor above the floor in which S lives.
1. Who among the following lives on the second floor of building A?

A. Z

B. W

C. Q

D. P

E. None of these

2. How many people live in building C?

A. One

B. Two

C. Three

D. Four

E. Five

3. Which of the following statement(s) is/are correct?

A. Q lives on the 5th floor.

B. W lives on the 8th floor.

C. T lives immediately below Y’s floor.

D. All are correct.

E. None is correct.

4. How many persons live between the floor on which Z and P live?

A. One

B. Two

C. Three

D. Four

E. None

5. How many floors are there in building A, B and C respectively?

A. 10, 10, 10

B. 10, 8, 8

C. 11, 8, 8

D. 12, 7, 9

E. 12, 9, 7

6. The question given below consists of a statement, followed by three arguments numbered I, II and III. You have to decide which of the given arguments is/are ‘strong’ arguments and which is/are ‘weak’ arguments and accordingly choose your answer from the alternatives given below each question.
Statement:
A shortage of bank branches and ATMs across India’s hinterland is holding back Prime Minister Narendra Modi’s financial inclusion efforts and risks angering rural voters ahead of elections next year. After taking office in 2014, Modi set an ambitious target to open a bank account for every household to ensure welfare funds flow directly to India’s poor, while improving access to credit and insurance programs. He pushed policies that helped bring 310 million people into the formal banking system in just four years, according to the World Bank. Based on the arguments stated below and the information stated above, which of the following arguments state the reason for the problem, ‘But many of India’s villages still lack bank branches or ATMs to help service new customers, while the pace of building new financial infrastructure has actually slowed’.
Arguments:
I. Because Modi’s government effectively forced poor citizens into the banking system by linking some welfare benefits to bank accounts, villagers have ended up stuck in long queues and struggling with ATMs that often run out of cash or break down.
II. With an election due next year, the mismatch between the government’s policies and the rural banking system is generating frustration among a key slice of India’s electorate.
III. The banking system struggled to keep up, while some gains proved temporary. Nearly half of the Indian bank accounts were inactive in 2017, meaning they weren’t used at all in the last 12 months

A. None is strong.

B. All except III and II are strong.

C. All except II and I are strong.

D. Only II is strong.

E. All are strong.

7. The question given below consists of a statement, followed by three arguments numbered I, II and III. You have to decide which of the arguments is/are ‘strong’ arguments and which is/are ‘weak’ arguments and accordingly choose your answer from the alternatives given below each question.
Statement:
World’s largest beer maker Heineken NV, the majority partner of United Breweries with Vijay Mallya, is understood to have sought legal opinion over its right to appoint a chairman at the Indian company. Heineken and some of its advisers believe that the shareholder agreement between Mallya and the beer giant has become null and void after India’s Enforcement Directorate attached his shares as part of its legal action against the liquor baron. Which among the following arguments support the above statement in the best possible manner?
Arguments:
I. UBL has stopped sharing confidential information with Mallya and has said that he is no longer privy to any strategic developments.
II. The board of UBL, India’s biggest beer company, had asked Mallya to either step down or appoint a nominee after the Securities and Exchange Board of India barred wilful defaulters from holding key board positions last year.
III. The company is functioning well and operations are in good shape but it is not good corporate governance to have an acting chairman for so long.

A. None is strong.

B. Only I and II are strong.

C. Only III is strong.

D. All except III and II are strong.

E. All are strong.

8. The Pakistani Army and other establishments are rallying behind a controversial dam project on the Indus River in disputed territory that has taken on new wind after India's frustration on attempts by Islamabad to get international funding for years. The issue is set to snowball into a controversy with India opposing the project as it falls in Pakistan Occupied Kashmir (PoK). Potential water wars between the two nations are a reality, with tensions flaring up after the 2016 Uri attack that left 19 soldiers dead. Prime Minister Narendra Modi has been vocal on water-sharing pacts between the nations and had famously told a review meeting of the Indus Waters Treaty in 2016 after the Uri attack that ‘rakt aur paani ek saath nahin beh sakta’ (blood and water cannot flow together). Which of the following is a logical corollary of the statement above?

A. Several government controlled institutions have made similar pledges, while appeals are also being made to overseas Pakistanis to donate money for the dam.

B. Pakistan has been trying unsuccessfully for years to raise funds from the World Bank and Asian Development Bank but facing frustration after the bodies stated that it was on disputed territory.

C. A more recent attempt to bring it under the China Pakistan Economic Corridor (CPEC) also did not go through after Beijing put up an ownership rights clause on the mega project.

D. Officers of the Pakistani armed forces are donating two days’ pay while soldiers have been mandated to donate one day’s pay for the project that is being touted as the solution to its water scarcity problems.

E. None can be deduced.

Direction (9-11): Study the following information to answer the given questions:
$ F 3 6 N @ 9 K T Q 5 C % 8 B # 7 D S * H 4 W L
Step 1 – Those number(s) which is/are immediately preceded by the symbol and immediately followed by an alphabet are written at the right end in ascending order.
Step 2 – After completing the step -1, Interchanging the odd number with the previous element in the series to form the step -2.
Step 3 – After completing the step – 2, Alphabet(s) which is/are immediately followed by a symbol are written between sixth and seventh element from the right end in alphabetical order from left to right.

9. Which of the following element is seventh to the right of the element which is tenth from the left end in step - 3?

A. B

B. *

C. N

D. W

E. H

10. If all the symbols are removed from the step -3, then which of the following element will be in the middle of the series?

A. H

B. B

C. C

D. Q

E. S

DOWNLOAD SBI PO Question Papers PDF

DOWNLOAD SBI CLERK Question Papers PDF

DOWNLOAD MORE BANK EXAMS E-BOOKS

Printed Study Material for SBI PO Exam

11. How many numbers are immediately followed and immediately preceded by an alphabet in step -3?

A. One

B. Two

C. Three

D. None

E. More than three

Direction (12-13): Study the following information to answer the given questions.
A, B, C and D are the corners of a rectangular field such that A is to the North of B. C is 0.2km to the East of A. The longer side of the rectangular field is 2km. E, F, G and H are the corners of an another rectangular field such that E is 1.6km to the west of G. H is 1.2km to the south of G and in the East of F. Rectangle field EFGH intersects the line joining AB and CD at KL and IJ respectively. It is also known that K is 0.9km towards the East of E and J is 0.4km towards the North of D. 12. If the person “X” wants to reach the point E from Point D, then which of the following will be the correct direction to reach his destination? A. X walks 1600m to the north of point D, then he take left turn and walks 1000m to reach point E B. X walks 1600m to the north of point D, then he take right turn and walks 500m to reach point and then he take right turn and walk 1200m and then he take right turn and walk 1600m to reach point E C. X walks 400m to the north from point D and then he take left turn and walk 200m and then he take right turn and walk 1200m and finally he take right turn and walk 900m to reach the point E D. X walks 400m to the north from point D and then he take left turn and walks 1100m and then he take left turn and walk 1200m to reach point E E. X walks 200m west from point D and then he takes right turn and walks 400m then he take left turn and walk 900m and finally he take right turn and walk 1200m to reach point E

13. What will be the minimum possible distance to reach point L from point C?

A. LJDBAC

B. LJHGIC

C. LJIKAC

D. LFEKAC

E. None of the above

Direction (14-16): Study the following information to answer the given questions.
A number arrangement machine when given an input line of numbers rearranges them following a particular rule. The following is an illustration of input and re-arrangement.
Input: 62 31 23 48 73 54 17 42
Step 1: 124 93 69 96 219 108 51 84
Step 2: 217 24 165 123 327 57 135
Step 3: 14 8 30 6 42 35 15
Step 4: 7 4 15 3 21 17.5 7.5
Step IV is the output of the above input.
As per the rules followed in the above steps, find out in each of the following questions the appropriate steps for the given input.
Input: 44 37 61 56 32 83 12 75

14. Which of the following element is second to the right of the element which is sixth from the right end in step III?

A. 20

B. 90

C. None of these

D. 32

E. 72

15. Which of the following are the first two elements in the last step?

A. 40.5, 45

B. 16, 4.5

C. 10, 36

D. 40.5, 7

E. 7, 45

16. What will be the resultant if second element from the left end in step 2 is multiplied by fourth element from the right end in step 3?

A. 2034

B. 2304

C. 7200

D. None of those given as option

E. 3362

Direction (17-21): Study the following information to answer the given questions:
Six persons are born in six different years. Their ages are calculated with respect to 2018. None of them is more than 90 years old. They like six different fruits Cherry, Orange, Apple, Mango, Banana and Grapes but not necessarily in the same order.
Note- The last two digits or reverse of the last two digits of the year in which a person is born can be the age of some other person. (Like A is born in 1943 and age of B is denoted by the last two digits of the birth year of A. Then B’s age will be either 34 or 43). Tarun was born in 1983.The difference between age of Tarun and the one who like cherry is 21 years. Visakha age was sum of all the digits of birth year of the one who like cherry. The one who like Mango is 5 years elder than Visakha. Only one person born was before Arko. Arko age was equal to the last two digit of the birth year of the one who likes Mango. The difference between ages of Arko and Viraj was 24years.The one who likes Banana born immediately before the one who like Grapes. The age of the one who likes Apple was not an even Number. Manika was elder than Charu.

17. Who among the following likes Orange?

A. Visakha

B. The one who was 56 years old

C. The one who like Banana

D. Charu

E. The one who was born in 2000

18. What is the age of the one who like Grapes and the one who like Apple respectively?

A. 7, 31

B. 70, 46

C. 35, 59

D. None of those given as options

E. 59, 35

19. What will be the sum of ages of the youngest and the eldest person in the group?

A. 101 years

B. 77 years

C. 111 years

D. 97 years

E. None of those given as options

20. Which of the following statement(s) is/are true?
I) Two persons were born between the one like Cherry and the one who like Orange.
II) The number of person born before Manika was same as the number of person born after Tarun.
III) The sum of digits of birth year of Viraj is not equal to Visakha age.

A. Only (I)

B. Only (II) and Only (III)

C. All (I), (II) and (III)

D. Only (I) and (II)

E. Only (III)

21. Arko like which of the following fruits?

A. Orange

B. Grapes

C. None of those given as options

D. Cherry

E. Banana

22. Popcorn may be a humble snack for movie goers but for India's multiplex industry it is the diet on which its revenue gets the much-needed bump even when movies don't fire the bottom line. Food and beverage (F&B) segment has been a high earner at nearly 25% for multiplexes, and that's why allowing food from outside can dim the prospects for this growing sector. Which of the following can be concluded based on the information provided above?

A. The end of prohibition on outside food will come as a big dampener just when the multiplex industry is growing rapidly and exploring newer sources of income.

B. If state governments force multiplexes to sell F & B items at maximum retail price and allow outside food, the industry may have to put up with lower earnings growth in the coming months. Shares of leading multiplexes saw a sharp fall on Friday.

C. For instance, a tub of popcorn in most multiplexes costs nearly two times the price of a movie ticket. This fact also reflects in the financial numbers of many multiplex companies.

D. As more multiplexes come up, owners look for newer sources of revenue.

E. All can be inferred.

23. Direction: Two statements are followed by three Conclusions I, II and III. You have to consider the statements to be true, even if they seem to be at variance from commonly known facts. You are to decide which of the given conclusions can definitely be drawn from the given statements and indicate your answer accordingly.
Statements:
I. Milk supply in Maharashtra was badly hit after thousands of dairy farmers launched a protest on Monday, demanding better price and subsidy of Rs 5 per litre. Tankers supplying milk to major cities including Mumbai, Pune,Nagpur, Nashik and others were blocked early morning leading to sudden crisis.
II. The agriculture ministry had mooted a proposal asking railways to make milk available at railway stations and at the meeting, sources said representatives from Amul India suggested that the national transporter could provide stalls to dairy companies or cooperatives where they could sell their products.
Conclusions:
I. Representatives of Amul India and food safety body FSSAI are willing to discuss issues and concerns of the dairy sector in the backdrop of the farmers agitation in Maharashtra that had led to severe shortage of milk in the state.
II. The farmers' groups and Maharashtra Kisan Sabha, are demanding a GST waiver on butter and milk powder apart from the subsidy.
III. Railways is drawing up a plan to make milk available at railway stations across the country to promote its consumption.

A. Only conclusion 3 follows

B. Only conclusion 1 follows

C. Either conclusion 3 or 2 follows

D. Neither conclusion 1 nor 3 follows

E. Both conclusions 1 and 2 follow

Direction (24-25): Study the following information carefully and answer the question given below:
A certain number of persons sit in a row adjacent to each other. Some of them like different fruits and others like different flowers. Six persons sit between the one who like mango and the one who like apple. The one who likes rose sit third to the left of the one who likes apple. Two persons sit between the one who likes rose and the one who likes marigold, who is not a neighbor of the one who likes mango. Only one person sit to the right of the one who likes marigold. No one sit to the right of the one who likes mango.
Note: The one who likes fruits faces South and the one who likes flowers face North.

24. How many persons sit in the row?

A. Fifteen

B. Seventeen

C. Thirteen

D. Eighteen

E. Eleven

25. What is the position of the person who likes Marigold with respect to the person who likes Apple?

A. Sixth to the right

B. Fifth to the right

C. Sixth to the left

D. Fifth to the left

E. Can’t be determined

DOWNLOAD SBI PO Question Papers PDF

DOWNLOAD SBI CLERK Question Papers PDF

DOWNLOAD MORE BANK EXAMS E-BOOKS

Printed Study Material for SBI PO Exam

Direction (26-27): Study the following information carefully and answer the question given below:
Five persons P, Q, R, S and T purchase some items one after other. Two persons bought between P and the one who bought Chair. Q purchase immediately before R but none of them bought Chair. Three persons bought between the one who purchase chair and the one who purchase table. Only two persons bought between the one who purchase wallet and the one who purchase table. Bag is purchased immediately before perfume. Only two persons bought between S and the one who purchase perfume. R does not buy wallet.

26. Who among the following purchase table?

A. P

B. T

C. R

D. Q

E. None of these

27. Which of the following statement(s) is/are correct?
I. R purchage Bag.
II. S doesn’t purchase Wallet.
III. T purchage Perfume.
IV. Q doesn’t purchage chair.

A. Only I and II

B. Only III and IV

C. Only I, II and III

D. Only I, II and IV

E. All of the above

Direction (28-29): Study the following information carefully and answer the given questions: 6 persons are celebrating their birthdays in first 6 months of a year (January- June). Rajat celebrates in the month of March. Shreya celebrate before Saroj. Only 1 person celebrates between Rajat and Suresh, who do not celebrate immediate before or immediate after Rajesh. Rajesh celebrates immediately before Rekha. Saroj celebrates neither in June nor immediately after Suresh.
28. In which month Shreya celebrates her birthday?

A. January

B. March

C. April

D. February

E. May

Direction (29-32): Read the following information carefully and answer the questions given below.
Q, W, E, R, T, Y, U, I and O are nine family members. Q is the father of E. Y is the daughter in law of W. T is the husband of Y. R is married to E. U is the only son of R. I is the granddaughter of Q. O is the son in law of T. R is the sister- in-law of T.

29. How is T related to U?

A. Maternal Uncle

B. Paternal Uncle

C. Father

D. Brother

E. None of these

30. Five trains namely Rajdhani express, Duranto express, Shatabdi express, Tejas express and Gareebrath express are running from Delhi to Dehradoon in different days of the week, starting from Monday to Friday. Two trains are running between Shatabdi and Duranto. Shatabdi is going to Dehradoon before Wednesday. Tejas express is going to Dehraddon immediate after Gareebrath. Rajdhani is not running on Friday. Then who among the following train are going to Dehradoon on Wednesday?

A. Rajdhanni

B. Duranto

C. Shatabdi

D. Tejas

E. Gareebrath

31. The Delhi High Court directed the Indian Air Force (IAF) to review and change its protocols dealing with issues of stress and substance abuse in the force, saying its systems have to be in tune with the law. The direction by a bench of justices S Muralidhar and Vinod Goel was issued as it disapproved of the manner in which the IAF had kept a 30-year-old corporal confined in a hospital's psychiatric ward as he was an alcoholic Which of the following can be logically inferred from the statement above?

A. Delhi High Court wants to make sure the IAF does not think that the Mental Health Act does not apply to it. There cannot be such a casual exercise that you can keep a person hospitalised for as long as you want.

B. The court was hearing a habeas corpus petition filed by the corporal's family, claiming their son was being illegally confined at the Army Base Hospital.

C. The court asked the IAF to deposit in court one complete copy of the Corporal's medical record by July 16 and listed the matter for further hearing on August 17

D. Central government standing counsel Ripudaman Singh Bhardwaj, appearing for the IAF, said the air force acted with bona fide intention in the instant matter. However, the court refused to accept the argument.

E. None can be inferred.

32. In what has been touted as the biggest Prime Day yet, Amazon India's 36-hour blockbuster sales event is finally here. Open exclusively for all its Prime members, this edition promises 200 new product launches from top brands, unparalleled deals and top-notch video releases. While all categories are proposing massive discounts as well as great payment offers, shoppers will have myriad options to choose from among large appliances like washing machines, refrigerators, televisions as well as ACs and microwave ovens. Which of the following is a perfect conjecture based on the given piece of information?

A. Other top deals include BPL's side by side refrigerator for a little over Rs 45,000 at no cost EMI and massive discounts on brands such as LG, Hitachi and Samsung.

B. Not just on TVs, Amazon Prime Day is handing out heavy discounts and deals in refrigerators as well.

C. Amazon Prime Day is giving shoppers a very unusual - and tempting - deal in majority of the categories. Prime members can get their hands on a great deal of products that too at massive discounts.

D. Moreover, Amazon promises to get it delivered as soon as July 19 - just three days from now.

E. None can be inferred.

33. In the question below, are given a statement followed by three courses of actions numbered I, II and III. On the basis of the information given, you have to assume everything in the statement to be true, and then decide which of the suggested courses of action logically follow(s) for pursuing.
Statement:
India’s secretive nuclear missile tracking ship, which will become part of an elaborate missile shield being planned against attacks is successfully undergoing harbour trials and is set to be delivered by December. The VC11184, a specialised Ocean Surveillance Ship being built for the National Technical Research Organisation (NTRO), is on its way to completion just over four years after it was ordered as part of the Modi government’s focus on creating a nuclear missile shield for the nation.
Courses of action:
I. The complex vessel, which will generate over 14 MW of power just to power up its tracking radars, will have multiple roles for India – from tracking enemy missiles to accurately giving data on tests that are routinely carried out of indigenous strategic missiles.
II. The next step will include sea trials by a joint team of the Indian Navy and the NTRO in which its specialised surveillance systems – three dome shaped antennas packed with sensors – will be extensively tested before the handing over.
III. The VC11184, which has not been given a formal ‘commissioning’ name yet, is one of the largest warships to be built at an Indian yard, weighing in at over 15,000 tonnes.

A. Only III follows

B. Neither I nor III follows

C. None follows

D. All except III and II

E. All I, II and III follow

34. The world’s largest online paid TV network Netflix may look at tweaking its premium strategy for India, as it finds itself lagging behind Hotstar and YouTube in the booming over-the-top (OTT) market. The US-headquartered company, which currently derives most of its streaming revenue from outside the US, may enter lower priced segments to attract new subscribers. When asked about the Indian OTT market during the June-quarter earnings call. ‘We’re way behind YouTube, Hotstar. Those are really the leaders on internet’. Which of the following can be deduced from the above information?

A. In India, Netflix competes with a host of OTT players, including Amazon Prime Video and Star TV-run Hotstar that charge lower subscription fees.

B. Hotstar, for instance, follows a hybrid subscription model and charges viewers for only premium content. In contrast, Netflix, which has invested heavily in producing slick shows in regional languages for the Indian audience, addresses affluent viewers at a hefty premium.

C. India will enter the top 10 largest global OTT video markets by 2022 with revenue of over Rs 5,500 crore.

D. A market like India has three-or-four different sort of growth patterns due to different demographics and segments and Netflix may address them one after the other.

E. None can be inferred.

35. Direction: The passage given below is followed by a question. Select the best answer from the choices given below. In the 1980s, 85 per cent of the total workforce of Tamil Nadu was employed outside the state. This percentage has seen a steady decline ever since then and in 2010 stumbled to 30 per cent. If the facts given in these statements are correct pertaining to the changes in the Tamil Nadu's workforce between 1980 and 2010, which of the following statement could be incorrect regarding the same.

A. Before 1980 people in Tamil Nadu were encouraged to learn Hindi but after 1980 the government has banned the teaching of Hindi in its schools.

B. Tamil Nadu has been creating more and more jobs by starting more and more factories in both the private and government sector.

C. The others states have started adopting the strategy of giving jobs to ‘sons of the soil’ and not those coming from outside the state.

D. The government of Tamil Nadu has debarred its people from leaving the state to exploit employment opportunities and has fortified the state at length to strengthen it.

E. None of these.

Direction (36-40): The chief minister of a state has to select 5 ministers in his cabinet: Abhishek, Bardhan, Chetan, Darshan, Edwin, Fawad, Gurdeep and Harsh. The age (in years) of each minister is different among 36, 38, 39, 45, 48, 54, 57 and 59 years. From the eight ministers, the CM has to select at least one minister who is in his thirties, at least one minister who is in his forties and at least two ministers who are in their fifties. The CM must ensure the following additional conditions are also met:
* Bardhan, who is in his thirties, will be in the cabinet only if Harsh is not in the cabinet.
* Fawad is younger than Harsh and Harsh will be in the cabinet only if Abhishek is in the cabinet.
* If Chetan is in the cabinet, Darshan must not be in the cabinet.
* Edwin is six years older than Chetan and if one of them is in the cabinet, the other must be in the cabinet.
* Gurdeep is three years younger than Abhishek.

* Darshan will be in the cabinet if and only if Fawad, who is in his fifties, is in the cabinet.
* If Edwin is in the cabinet, Harsh must be in the cabinet.

36. In how many ways can the cabinet be selected?

A. 1

B. 2

C. 3

D. 4

E. 6

37. Who among the following cannot be in the cabinet?

A. Bardhan

B. Chetan

C. Fawad

D. Gurdeep

E. Abhishek

Direction (38-42): In the following question, a given question is followed by information in three statements. You have to decide the data in which statement(s) is sufficient to answer the question and mark your answer accordingly.
38. Six persons namely viz. A, B, C, D, E and F bought six different branded shoes namely viz. Adidas, Fila, Lotto, Nike, Puma and Reebok but not necessarily in same order. Each of these shoes worth of different amounts among Rs.1700, Rs.2200, Rs.2500, Rs.2700, Rs.3000 and Rs.3600. Who bought Nike shoes and how much does they cost?
I. E bought Lotto shoes. C did not buy Fila shoes. A did not buy Nike shoes. The difference in cost of shoes that A bought and Adidas shoes is Rs.800.
II. F bought Reebok shoes, which costs Rs.3600. D did not but Adidas shoes or Puma shoes. The cost of Puma shoes is Rs.2500. The cost of Nike shoes is more than that of A and C’s shoes. The cost of shoes that C bought is more than that of E’s shoes.
III. D bought shoes for Rs.2700, which were not of Fila. A and C did not buy Adidas shoes. E bought shoes worth lowest price. The cost of C’s shoes is more than that of A’s shoes but less than F’s shoes. F bought Reebok shoes.

A. All the statements are required.

B. Only I and II are sufficient.

C. Only II is sufficient.

D. Either I and II or I and III are sufficient.

E. Insufficient data.

Direction (39-42): In the following question, a given question is followed by information in three statements. You have to decide the data in which statement(s) is sufficient to answer the question and mark your answer
accordingly.
39. Karan walks from point P towards east to reach a point Q at a distance of 6m from P, then he moves towards south and walks a distance of 8m to reach a point R. From there he moves towards east a distance of 5m and reaches a point S. Then he moves towards north a distance of 4m to point T. From there he moves towards east a distance of 5m to reach a point U. Then he moves towards north a distance of 10m to point V. From there he moves towards east a distance of 6m to reach a point W. Is the shortest distance between points Q and X an integer value?
I. From point W, the person moves towards south and walks for 14m till point X.
II. From point W, the person moves towards south and walks for 7m till point Y, then he move towards south-east direction for 8m and reach point X.
III. From point W, the person moves in west direction for 16m and stopped at point Y, he then moves in north direction for 6m till point Z. From point Z, he moves 5m in east direction till point X.

A. All the statements are required.

B. Only I and II alone are sufficient.

C. Only II is sufficient.

D. Only I and III alone are sufficient.

E. Insufficient data.

Direction (40-44): The following questions consists of a question followed by three statements I, II and III. You have to decide whether the data given in the statements are sufficient for answering the question. Read all the statements carefully and seek all the possible combinations which could be sufficient for answering the question.

40. 8 friends A, B, C, D, E, F, G and H are sitting around a circular table in such a way that some of them faces towards the centre while others faces away from the centre. Each of them likes different colors- Blue, Black, Green, Orange, Pink, Purple, Red and White. Who sits to the immediate left of one who likes blue?

I. B sits immediate left of C, who sits third to the right of one who likes Pink. G likes white and sits second to the left of one who likes Red, who sits third to the right of E. A sits second to the right of F, who sits third to the left of G. F sits third to the right of D.
II. F faces outside and H, who likes Red sits immediate right of F.B, who likes Green sits second to the left of F, who sits third to the right of D. E sits second to the left of A, who sits second to right of one who likes Black, who sits
immediate right of one who likes Red.
III. One who likes Purple sits second to the right of one who likes Green, who sits immediate right of D, who faces away from the center. H sits second to the left of G, who sits third to the right of one who likes Black. C likes Orange and sits third to right of one who likes Pink, who sits second to the right of F.

A. All I, II and III

B. Any 2 of them

C. Both I and II

D. Both II and III

E. Both I and III

Direction (41-45): In the following question, a given question is followed by information in three statements. You have to decide the data in which statement(s) is sufficient to answer the question and mark your answer accordingly.

41. A, B, C, D, E and F are six employees. Each of them gets different number of mails–15,20,30, 50, 60 and 70 everyday but not necessarily in the same order. Who is the shortest among all and what is the number of mails that D gets?
I. E is taller than only D and F. C is the tallest among all but does not get the least number of mails. A gets half the number of mails of what B gets.
II. A is taller than B. The sum of mails of A and E is equal to the number of mails that D gets. The one, who gets 50 mails is not the shortest. D does not get the highest number of mails.
III. A does not get 15 mails. B is not taller than A. D gets more mails than E but less than C.

A. Statement I and II together are sufficient.

B. Statement I alone or statement II alone is sufficient.

C. Statement II alone is sufficient.

D. Statement I and II together are not sufficient.

E. Statement I alone is sufficient.

42. Direction: Each of the questions below consists of a question and four statements numbered I, II, III and IV given below it. You have to decide whether the data provided in the statements are sufficient to answer the question. Read all the three statements and give answer:
What is the code for ‘AM’ in the given code language?
I. “AB CD EF” is written as ’12 34 56’
II. “BB FG IK” is written as ‘22 67 911’
III. “AC BD DF” is written as ‘13 24 46’
IV. “CD EK LA” is written as ’34 511121’

A. If the data in Statement I and II are sufficient to answer the question while the data in Statement III and IV is not required to answer the question

B. If the data in Statement I and III are sufficient to answer the question, while the data in Statement II and IV is not required to answer the question

C. If the data in Statement I and IV are sufficient to answer the question, while the data in Statement II and III is not required to answer the question

D. If the data in either Statement I alone or Statement II alone or Statement III or statement IV alone is sufficient to answer the question

E. If the data in all the Statement I, II, IV and III together are necessary to answer the question

Direction (43-45): Study the information carefully and answer the questions given below.
Six persons sit around a triangular table such that three of them sit at the corners and the rest on the middle of the side. The one who sits at the corner face away from the center and the one who sit at the middle of the side face towards the center. They also like different colours i.e. black, brown, blue, pink, purple and cyan. The one who likes black colour sits at the corner. The one who likes cyan colour sits immediate right to the one who likes purple colour. C sits second to the right to B, who likes blue colour. D faces the center and sits 3rd right to the one who

43. Who among the following likes purple colour?

A. E

B. F

C. C

D. B

E. None of these

44. Who among the following sits 2nd to the right of the one who is 3rd to the left of A?

A. E

B. F

C. C

D. B

E. none of these

45. What is the position of C with respect to the one who likes black colour?

A. Immediate right

B. Immediate left

C. Second to the left

D. Second to the right

E. None of these

DOWNLOAD SBI PO Question Papers PDF

DOWNLOAD SBI CLERK Question Papers PDF

DOWNLOAD MORE BANK EXAMS E-BOOKS

Printed Study Material for SBI PO Exam

<<Go Back To Main Page